Last visit was: 24 Apr 2024, 14:11 It is currently 24 Apr 2024, 14:11

Close
GMAT Club Daily Prep
Thank you for using the timer - this advanced tool can estimate your performance and suggest more practice questions. We have subscribed you to Daily Prep Questions via email.

Customized
for You

we will pick new questions that match your level based on your Timer History

Track
Your Progress

every week, we’ll send you an estimated GMAT score based on your performance

Practice
Pays

we will pick new questions that match your level based on your Timer History
Not interested in getting valuable practice questions and articles delivered to your email? No problem, unsubscribe here.
Close
Request Expert Reply
Confirm Cancel
SORT BY:
Date
Tags:
Difficulty: 555-605 Levelx   Statistics and Sets Problemsx                              
Show Tags
Hide Tags
GMAT Club Legend
GMAT Club Legend
Joined: 19 Dec 2014
Status:GMAT Assassin/Co-Founder
Affiliations: EMPOWERgmat
Posts: 21846
Own Kudos [?]: 11665 [0]
Given Kudos: 450
Location: United States (CA)
GMAT 1: 800 Q51 V49
GRE 1: Q170 V170
Send PM
Senior Manager
Senior Manager
Joined: 25 Aug 2020
Posts: 252
Own Kudos [?]: 116 [0]
Given Kudos: 218
Send PM
GMAT Club Legend
GMAT Club Legend
Joined: 08 Jul 2010
Status:GMAT/GRE Tutor l Admission Consultant l On-Demand Course creator
Posts: 5957
Own Kudos [?]: 13387 [0]
Given Kudos: 124
Location: India
GMAT: QUANT+DI EXPERT
Schools: IIM (A) ISB '24
GMAT 1: 750 Q51 V41
WE:Education (Education)
Send PM
Tutor
Joined: 17 Jul 2019
Posts: 1304
Own Kudos [?]: 2285 [1]
Given Kudos: 66
Location: Canada
GMAT 1: 780 Q51 V45
GMAT 2: 780 Q50 V47
GMAT 3: 770 Q50 V45
Send PM
Re: If Q is an odd number and the median of Q consecutive integers is 120 [#permalink]
1
Kudos
Expert Reply
Video solution from Quant Reasoning:
Subscribe for more: https://www.youtube.com/QuantReasoning? ... irmation=1
Director
Director
Joined: 04 Jun 2020
Posts: 552
Own Kudos [?]: 67 [0]
Given Kudos: 626
Send PM
Re: If Q is an odd number and the median of Q consecutive integers is 120 [#permalink]
BrentGMATPrepNow wrote:
Walkabout wrote:
If Q is an odd number and the median of Q consecutive integers is 120, what is the largest of these integers?

(A) (Q - 1)/2 + 120
(B) Q/2 + 119
(C) Q/2 + 120
(D) (Q + 119)/2
(E) (Q + 120)/2


A very fast solution is to see what happens when Q = 1.
This means that there's only ONE integer in the set.
So, if the median of the set is 120, then the set is {120}, which means the greatest value in the set is 120

So the correct answer choice should yield 120 when Q = 1.

a) (1-1)/2 + 120 = 120 PERFECT!
b) 1/2 + 119 = some non-integer
c) 1/2 + 120 = some non-integer
d) (1+119)/2 = 60
e) (1+120)/2 = some non-integer

Since only answer choice A yield the correct output, it is the correct answer.

Cheers,
Brent


BrentGMATPrepNow
This is very helpful! How do we know not to test another number (e.g., when Q=3)? Overall, I am confused on the rules pertaining to how to know if you need to test more smart numbers, or do you only need to choose one set of smart numbers if they satisfy all the conditions? Thank you in advance :)
GMAT Club Legend
GMAT Club Legend
Joined: 12 Sep 2015
Posts: 6821
Own Kudos [?]: 29907 [2]
Given Kudos: 799
Location: Canada
Send PM
Re: If Q is an odd number and the median of Q consecutive integers is 120 [#permalink]
2
Kudos
Expert Reply
Top Contributor
woohoo921 wrote:
BrentGMATPrepNow
This is very helpful! How do we know not to test another number (e.g., when Q=3)? Overall, I am confused on the rules pertaining to how to know if you need to test more smart numbers, or do you only need to choose one set of smart numbers if they satisfy all the conditions? Thank you in advance :)


The key here is that each question on the GMAT has exactly one correct answer.
So, it can't be the case that, when Q = 1, the correct answer is A, but when Q = 3, the correct answer is to something else.
Intern
Intern
Joined: 28 Mar 2021
Posts: 23
Own Kudos [?]: 14 [0]
Given Kudos: 48
Schools: HBS '25 (A)
Send PM
Re: If Q is an odd number and the median of Q consecutive integers is 120 [#permalink]
Testing cases is the best option here, but here is the algebra for those who are interested:
S = smallest integer, L = largest integer, Q=number of terms in a set (and we're given that it's odd).

In any set of consecutive integers, the mean is equal to the median and either can be found by finding the average of the smallest and the largest integer. Since median is 120, it follows that:
\(\frac{(S+L)}{2} = 120\)
\(S+L = 240\)
\(S = 240-L\)

The number of terms (inclusive) in a set in which the difference between each term is constant is given by:
\(\frac{(L-S)}{(Difference)} + 1\)
Since we have a set of consecutive integers, the difference between each term is 1 and so:

\(Q=(L-S)+1 \)
\(L = Q-1+S \) (Plug in for S using above)
\(L=Q-1+240-L\)
\(2L=Q-1+240\)
\(L=\frac{(Q-1)}{2} + 120\)
Director
Director
Joined: 04 Jun 2020
Posts: 552
Own Kudos [?]: 67 [0]
Given Kudos: 626
Send PM
Re: If Q is an odd number and the median of Q consecutive integers is 120 [#permalink]
BrentGMATPrepNow wrote:
woohoo921 wrote:
BrentGMATPrepNow
This is very helpful! How do we know not to test another number (e.g., when Q=3)? Overall, I am confused on the rules pertaining to how to know if you need to test more smart numbers, or do you only need to choose one set of smart numbers if they satisfy all the conditions? Thank you in advance :)


The key here is that each question on the GMAT has exactly one correct answer.
So, it can't be the case that, when Q = 1, the correct answer is A, but when Q = 3, the correct answer is to something else.





Thank you! To confirm, the only time you would need to choose another smart number or set of smart number(s) to test is when you have more than one answer choice that has the same value? Otherwise, if you get a unique value, you can just move on?
GMAT Club Legend
GMAT Club Legend
Joined: 19 Dec 2014
Status:GMAT Assassin/Co-Founder
Affiliations: EMPOWERgmat
Posts: 21846
Own Kudos [?]: 11665 [1]
Given Kudos: 450
Location: United States (CA)
GMAT 1: 800 Q51 V49
GRE 1: Q170 V170
Send PM
Re: If Q is an odd number and the median of Q consecutive integers is 120 [#permalink]
1
Kudos
Expert Reply
woohoo921 wrote:
BrentGMATPrepNow wrote:
woohoo921 wrote:
BrentGMATPrepNow
This is very helpful! How do we know not to test another number (e.g., when Q=3)? Overall, I am confused on the rules pertaining to how to know if you need to test more smart numbers, or do you only need to choose one set of smart numbers if they satisfy all the conditions? Thank you in advance :)


The key here is that each question on the GMAT has exactly one correct answer.
So, it can't be the case that, when Q = 1, the correct answer is A, but when Q = 3, the correct answer is to something else.





Thank you! To confirm, the only time you would need to choose another smart number or set of smart number(s) to test is when you have more than one answer choice that has the same value? Otherwise, if you get a unique value, you can just move on?


Hi woohoo921,

Yes - when TESTing VALUES, if only one of the 5 answers matches what you are looking for, then that answer is the correct answer. If more than one answer matches, then one of those answers is the correct answer (and the other(s) only sometimes match what you are looking for) - meaning that you would then have to TEST again with a different value (or values) to find the one answer that is ALWAYS a match.

GMAT assassins aren't born, they're made,
Rich

Contact Rich at: Rich.C@empowergmat.com
Tutor
Joined: 11 May 2022
Posts: 1092
Own Kudos [?]: 697 [1]
Given Kudos: 81
Send PM
Re: If Q is an odd number and the median of Q consecutive integers is 120 [#permalink]
1
Kudos
Expert Reply
EMPOWERgmatRichC wrote:
woohoo921 wrote:



Thank you! To confirm, the only time you would need to choose another smart number or set of smart number(s) to test is when you have more than one answer choice that has the same value? Otherwise, if you get a unique value, you can just move on?


Hi woohoo921,

Yes - when TESTing VALUES, if only one of the 5 answers matches what you are looking for, then that answer is the correct answer. If more than one answer matches, then one of those answers is the correct answer (and the other(s) only sometimes match what you are looking for) - meaning that you would then have to TEST only the answer choices that are still remaining (you do not need to test anything that was already eliminated with your first set of values) again with a different value (or values) to find the one answer that is ALWAYS a match.

GMAT assassins aren't born, they're made,
Rich

Contact Rich at: Rich.C@empowergmat.com


woohoo921

Maybe more cooks in the kitchen than you need, especially since Rich and Brent have already done a great job answering...great enough that I'll simply add a few words to Rich's post (in red) juuuuuust in case it wasn't already clear.
Intern
Intern
Joined: 06 Jun 2018
Posts: 24
Own Kudos [?]: 13 [0]
Given Kudos: 666
Send PM
Re: If Q is an odd number and the median of Q consecutive integers is 120 [#permalink]
If we were to derive it.
nth term in Arithmetic Progression is

\(t_n = a + (n-1)d\)

where a is first term, n is number of the term in the sequence, d is difference between two successive terms and \(t_n\) is the actual value of nth term

120 is the median and Q is given as odd. So it is the \(\frac{(Q+1)}{2}\) th term. Here d is 1 since numbers are consecutive.

\(120 = a + (\frac{(Q+1)}{2} - 1) 1\)

\(120 = a + (\frac{(Q-1)}{2})\)

\(a = 120 - \frac{(Q-1)}{2}\)

We will use this value of a in the next step.

Now the Qth term which will be the largest term in the sequence

\(= a + (Q-1) 1\)

\(= 120 - \frac{(Q-1)}{2} + (Q-1)\)

\(= 120 + \frac{(Q-1)}{2}\)

Ans: A
GMAT Club Bot
Re: If Q is an odd number and the median of Q consecutive integers is 120 [#permalink]
   1   2 
Moderators:
Math Expert
92902 posts
Senior Moderator - Masters Forum
3137 posts

Powered by phpBB © phpBB Group | Emoji artwork provided by EmojiOne